TEST BANK management advisory services by tan t03 working capital finance

39 561 0
TEST BANK management advisory services by tan t03   working capital finance

Đang tải... (xem toàn văn)

Tài liệu hạn chế xem trước, để xem đầy đủ mời bạn chọn Tải xuống

Thông tin tài liệu

MANAGEMENT ADVISORY SERVICES WORKING CAPITAL FINANCE WORKING CAPITAL POLICY Which of the following statements is incorrect about working capital policy? (M) a A company may hold a relatively large amount of cash if it anticipates uncertain sales levels in the coming year b Credit policy has an impact on working capital since it has the potential to influence sales levels and the speed with which cash is collected c The cash budget is useful in determining future financing needs d Holding minimal levels of inventory can reduce inventory carrying costs and cannot lead to any adverse effects on profitability e Managing working capital levels is important to the financial staff since it influences financing decisions and overall profitability of the firm Brigham Cash Marketable securities Accounts receivable Inventories Net fixed assets Total assets As a company becomes more conservative with respect to working capital policy, it would tend to have a(n) a Increase in the ratio of current liabilities to noncurrent liabilities b Decrease in the operating cycle c Decrease in the quick ratio d Increase in the ratio of current assets to noncurrent assets CMA 1290 1-23 As a company becomes more conservative in its working capital policy, it would tend to have a(n) A Decrease in its acid-test ratio B Increase in the ratio of current liabilities to noncurrent liabilities C Increase in the ratio of current assets to units of output D Increase in funds invested in common stock and a decrease in funds invested in marketable securities CMA 1296 1-8 Ski Lifts Inc is a highly seasonal business The following summary balance sheet provides data for peak and off-peak seasons (in thousands of dollars): Peak Off-peak RPCPA, AICPA, CMA & CIA EXAMINATION QUESTIONS $ 30 20 20 50 500 $620 Spontaneous liabilities $ 30 $ 10 Short-term debt 50 Long-term debt 300 300 Common equity 310 310 Total claims $690 $620 From this data we may conclude that (M) a Ski Lifts has a working capital financing policy of exactly matching asset and liability maturities b Ski Lifts’ working capital financing policy is relatively aggressive; that is, the company finances some of its permanent assets with short-term discretionary debt c Ski Lifts follows a relatively conservative approach to working capital financing; that is, some of its short-term needs are met by permanent capital d Without income statement data, we cannot determine the aggressiveness or conservatism of the company’s working capital financing policy Brigham Conservative Working Capital Policy * Compared to other firms in the industry, a company that maintains a conservative working capital policy will tend to have a (D) a Greater percentage of short-term financing b Greater risk of needing to sell current assets to repay debt c Higher ratio of current assets to fixed assets d Higher total asset turnover RPCPA 0595 $ 50 40 100 500 $690 Aggressive Working Capital Policy * A firm following an aggressive working capital strategy would (M) a Hold substantial amount of fixed assets b Minimize the amount of short-term borrowing c Finance fluctuating assets with long-term financing d Minimize the amount of funds held in very liquid assets RPCPA 1091 The working capital financing policy that subjects the firm to the greatest risk of being unable to meet the firm’s maturing obligations is the policy that finances (E) a Fluctuating current assets with long-term debt b Permanent current assets with long-term debt c Permanent current assets with short-term debt d Fluctuating current assets with short-term debt CMA 1295 1-2 Page of 39 MANAGEMENT ADVISORY SERVICES WORKING CAPITAL FINANCE Since Marsh, Inc is experiencing a sharp increase in sales activity and a steady increase in production, the management of Marsh has adopted an aggressive working capital policy Therefore, the company's current level of net working capital A Would most likely be the same as in any other type of business condition as business cycles tend to balance out over time B Would most likely be lower than under other business conditions in order that the company can maximize profits while minimizing working capital investment C Would most likely be higher than under other business conditions so that there will be sufficient funds to replenish assets D Would most likely be higher than under other business conditions as the company's profits are increasing CMA 0689 1-11 Aggressive vs Conservative Working Capital Policy Clay Corporation follows an aggressive financing policy in its working capital management while Lott Corporation follows a conservative financing policy Which one of the following statements is correct? A Clay has a low ratio of short-term debt to total debt while Lott has a high ratio of shortterm debt to total debt B Clay has a low current ratio while Lott has a high current ratio C Clay has less liquidity risk while Lott has more liquidity risk D Clay's interest charges are lower than Lott's interest charges CMA 1284 1-22 RPCPA, AICPA, CMA & CIA EXAMINATION QUESTIONS Page of 39 MANAGEMENT ADVISORY SERVICES WORKING CAPITAL FINANCE WORKING CAPITAL MANAGEMENT Appropriate Level of Working Capital Determining the appropriate level of working capital for a firm requires (E) a Evaluating the risks associated with various levels of fixed assets and the types of debt used to finance these assets b Changing the capital structure and dividend policy for the firm c Maintaining short-term debt at the lowest possible level because it is ordinarily more expensive than long term debt d Offsetting the profitability of current assets and current liabilities against the probability of technical insolvency e Maintaining a high proportion of liquid assets to total assets in order to maximize the return on total investments CMA 0689 1-10, 0696 1-16, RPCPA 0596 Which of the following statements about current asset management is most correct? (E) a A positive net float means that a company has more cash available for its use than the amount shown in the company’s books b Use of a lockbox reduces the possibility that petty cash will be lost c Depreciation has an impact on the cash budget d Statements a and c are correct Brigham Working Capital Working Capital Defined * The amount of long-term capital that is made to revolve in conducting operations and serves as the lifeblood of the company (E) a Paid-up capital c Working capital b Net worth d None of these RPCPA 0588 The working capital is the difference between A Current assets and current liabilities C Total assets and total liabilities B Fixed assets and fixed liabilities D Equity and cash CMA 0692 1-25 Net Working Capital is the: A Difference between short-term assets and short term liabilities B Difference between long-term assets and long term liabilities C Difference between long-term assets and short term liabilities D None of the above RPCPA, AICPA, CMA & CIA EXAMINATION QUESTIONS B&M Effect of Transactions on Working Capital Starrs Company has current assets of $300,000 and current liabilities of $200,000 Starrs could increase its working capital by the (E) A Prepayment of $50,000 of next year's rent B Refinancing of $50,000 of short-term debt with long-term debt C Purchase of $50,000 of temporary investments for cash D Collection of $50,000 of accounts receivable CMA 1293 1-19 10 Starrs has current assets of 300,000 and current liabilities of 200,000 Starrs could increase its working capital by the A Prepayment of 50,000 of next year's rent B Refinancing of 50,000 of short-term debt with long-term debt C Acquisition of land valued at 50,000 through the issuance of common shares D Purchase of 50,000 of financial assets held for trading for cash CMA 1293 1-19 * Which of the following transactions causes an increase in working capital? a Sale of merchandise on credit at a price above cost b Sale of marketable securities at a price below cost c Collection of an account receivable d Return to supplier of defective merchandise purchased on credit Full credit allowed by supplier RPCPA 1075 Other things held constant, which of the following will cause an increase in working capital? (M) a Cash is used to buy marketable securities b A cash dividend is declared and paid c Merchandise is sold at a profit, but the sale is on credit d Long-term bonds are retired with the proceeds of a preferred stock issue e Missing inventory is written off against retained earnings Brigham 11 Which one of the following would increase the working capital of a firm? A Cash payment of payroll taxes payable B Purchase of a new plant financed by a 20-year mortgage C Cash collection of accounts receivable D Refinancing a short-term note payable with a two-year note payable 12 CMA 1294 1-15 If a firm increases its cash balance by issuing additional shares of common stock, working capital (E) a Remains unchanged and the current ratio remains unchanged Page of 39 MANAGEMENT ADVISORY SERVICES b Increases and the current ratio remains unchanged c Increases and the current ratio decreases d Increases and the current ratio increases WORKING CAPITAL FINANCE a Cash flow control b Cash surplus investment CMA 1294 1-30 13 The following transactions occurred during a company's first year of operations: I Purchased a delivery van for cash I Borrowed money by issuance of short-term debt III Purchased treasury stock Which of the items above caused a change in the amount of working capital? A I only C II and III only B I and II only D I and III only CIA 0593 IV-28 Comprehensive 14 All of the following statements in regard to working capital are correct except (M) a Current liabilities are an important source of financing for many small firms b Profitability varies inversely wit liquidity c The hedging approach to financing involves matching maturities of debt with specific financing needs d Financing permanent inventory buildup with long-term debt is an example of an aggressive working capital policy CMA 0696 1-29 CASH MANAGEMENT Function 15 When managing cash and short-term investments, a corporate treasurer is primarily concerned with (E) a Maximizing rate of return b Minimizing taxes c Investing in Treasury bonds since they have no default risk d Investing in common stock due to the dividend exclusion for federal income tax purposes e Liquidity and safety CMA 1295 I-12 Determining the appropriate target cash balance involves assessing the trade-off between: A Income and diversification B The benefit and cost of liquidity C Balance sheet strength and transaction needs D All of the above B&M * c Maximizing sales d Obtaining financing services RPCPA 0590 Firms would need to hold zero cash when: A Transaction-related needs are greater than cash inflows B Transaction-related needs are less than cash inflows C Transaction-related needs are not perfectly synchronized with cash inflows D Transaction-related needs are perfectly synchronized with cash inflows B&M 16 Which of the following statements is most correct? (E) a A good cash management system would minimize disbursement float and maximize collections float b If a firm begins to use a well-designed lockbox system, this will reduce its customers’ net float c In the early 1980s, the prime interest rate hit a high of 21 percent In 2000 the prime rate was considerably lower That sharp interest rate decline has increased firms’ concerns about the efficiency of their cash management programs d If a firm can get its customers to permit it to pay by wire transfers rather than having to write checks, this will increase its net float and thus reduce its required cash balances e A firm that has such an efficient cash management system that it has positive net float can have a negative checkbook balance at most times and still not have its checks bounce Brigham Motives for Holding Cash 17 According to John Maynard Keynes, the three major motives for holding cash are for A Transactional, psychological, and social purposes B Speculative, fiduciary, and transactional purposes C Speculative, social, and precautionary purposes D Transactional, precautionary, and speculative purposes CMA 1286 1-32 * A precautionary motive for holding excess cash is (E) a To enable a company to meet the cash demands from the normal flow of business activity b To enable a company to avail itself of a special inventory purchase before prices rise to higher levels c To enable a company to have cash to meet emergencies that may arise periodically d To avoid having to use the various types of lending arrangements available to cover projected cash deficits RPCPA 0595 Which of the following is not a major function in cash management? (E) RPCPA, AICPA, CMA & CIA EXAMINATION QUESTIONS Page of 39 MANAGEMENT ADVISORY SERVICES WORKING CAPITAL FINANCE 18 The amount of cash that a firm keeps on hand in order to take advantage of any bargain purchases that may arise is referred to as its A Transactions balance C Precautionary balance B Compensating balance D Speculative balance CIA 1194 IV-15 25 Which of the following statements concerning zero balance accounts is not correct? A They are set up to handle disbursement activity B The account has a minimum amount at all times C Checks are automatically transferred into the account as checks presented for payment D The transfer is automatic and involves an accounting entry only E The master and the zero balance account locate at the same bank B&M 26 Which of the following is used to control disbursements? A Concentration banking C Lock-box system B Zero-balance account D Fedwire B&M 19 All of the following are valid reasons for a business to hold cash and marketable securities except to A Satisfy compensating balance requirements B Maintain adequate cash needed for transactions C Meet future needs D Earn maximum returns on investment assets CMA 0694 1-22 Which of the following statements is most correct? (M) a The cash balances of most firms consist of transactions, compensating, precautionary, and speculative balances The total desired cash balance can be determined by calculating the amount needed for each purpose and then summing them together b The easier a firm’s access to borrowed funds the higher its precautionary balances will be, in order to protect against sudden increases in interest rates c For some firms, holding highly liquid marketable securities is a substitute for holding cash because the marketable securities accomplish the same objective as cash Brigham d Firms today are more likely to rely on cash than on reserve borrowing power or marketable securities for speculative purposes because of the need to move quickly 12 A large firm may hold substantial cash balances because: A These balances are required by the bank B The company may have accounts in many different banks C The company may have a very decentralized organization D All of the above RPCPA, AICPA, CMA & CIA EXAMINATION QUESTIONS B&M 13 Most large firms hold a cash balance greater than most models imply because: A It is too difficult to estimate the costs of security transactions B Banks are compensated by account balances for payment of services C Corporations have few bank accounts and it is difficult to manage their cash D Cash is costless and need not be managed closely B&M 14 We should expect cash balances to increase when: A The transaction costs of buying or selling interest-bearing securities increase B Interest rates increase C Sales volume falls D Uncertainty about day-to-day or week-to-week cash flows decreases B&M Methods of Accelerating Cash Collections 32 Banks generally offer the following cash management services: A Processing checks C Running lock-boxes B Transferring funds D All of the above B&M 33 Banks generally offer the following services: A Processing checks B Running lock-boxes B&M C Providing advice and references D All of the above Disbursement Float 15 Checks written by the firm are said to generate: A Availability float C Disbursement float B Ledger float D Book float B&M 16 The difference between bank cash and book cash is called: A Disbursement float C Availability float B Net float D None of the above B&M Maximizing Net Float 20 Which of the following statements is most correct? (E) a A cash management system that minimizes collections float and maximizes disbursement float is better than one with higher collections float and lower disbursement float b A cash management system that maximizes collections float and minimizes disbursement float is better than one with lower collections float and higher disbursement float c The use of a lockbox is designed to minimize cash theft losses If the cost of the lockbox is less than theft losses saved, then the lockbox should be installed Brigham Page of 39 MANAGEMENT ADVISORY SERVICES WORKING CAPITAL FINANCE d Other things held constant, a firm will need a smaller line of credit if it can arrange to pay its bills by the 5th of each month than if its bills come due uniformly during the month Which of the following statements is most correct? (M) a Poor synchronization of cash flows that results in high cash management costs can be partially offset by increasing disbursement float and decreasing collections float b The size of a firm’s net float is primarily a function of its natural cash flow synchronization and how it clears its checks c Lockbox systems are used mainly for security purposes as well as to decrease the firm’s net float d If a firm can speed up its collections and slow down its disbursements, it will be able to reduce its net float e A firm practicing good cash management and making use of positive net float will bring its check book balance as close to zero as possible, but must never generate a negative book balance Brigham Draft 22 A working capital technique that delays the outflow of cash is (E) A Factoring C A lock-box system B A draft D Electronic funds transfer CMA 1293 1-21 23 A working capital technique that increases the payable float and therefore delays the outflow of cash is (E) A Concentration banking C Electronic Data Interchange (EDI) B A draft D A lockbox system CMA 1296 1-5 Automated Clearing House (ACH) Electronic Transfer 27 Electronic cash transfers offer several advantages, including: A A low marginal transactions cost C Easy automation of record-keeping B A reduced float D All of the above B&M 31 A check processed through ACH (Automated Clearing House): A Will clear immediately C Will take two or three days to clear B Will clear the same day D None of the above B&M Lock-Box System 21 The most common cash management technique used to speed up collections is: A Concentration banking C Lock-boxes B Wire transfers D In-house processing B&M 23 By getting closer to the source of payment, lock-boxes can be used to reduce: A Availability or clearing float C In-house processing float B Mail float D Disbursement float B&M 24 An automated clearing house (ACH) electronic transfer is a(n) a Electronic payment to a company’s account at a concentration bank b Check that must be immediately cleared by the Federal Reserve Bank c Computer-generated deposit ticket verifying deposit of funds CMA 0694 1-23) d Check-like instrument drawn against the payor and not against the bank 21 A lock-box system A Reduces the need for compensating balances B Provides security for late night deposits C Reduces the risk of having checks lost in the mail D Accelerates the inflow of funds A lockbox plan is most beneficial to firms that (M) a Send payables over a wide geographic area b Have widely disbursed manufacturing facilities c Have a large marketable securities account to protect d Hold inventories at many different sites e Make collections over a wide geographic area RPCPA, AICPA, CMA & CIA EXAMINATION QUESTIONS CMA 1293 1-20 Brigham Page of 39 MANAGEMENT ADVISORY SERVICES * WORKING CAPITAL FINANCE The following practices will impact the cash flow of the company: (E) a Sales personnel are unequivocally responsible for collecting their credit sales b Sales commissions are based on collected invoices c Statement of accounts receivable are reconciled with customers and regularly sent for confirmation RPCPA 0594 d Automatic transfer of funds is arranged with banks regarding deposits of branches Wire Transfer 28 The fastest but most expensive way to transfer surplus funds from the local deposit bank to the concentration bank is: A A lock-box system D An in-house processing float system B A mail float system E An availability float system C A wire transfer B&M Concentration Banking 22 Which of the following is a way for companies to speed up collections? A Remote disbursing C Baumol model B Concentration banking D All of the above B&M Comprehensive * Which of the following actions would not be consistent with good management? (M) a Increased synchronization of cash flows b Minimize the use of float c Maintaining an average cash balance equal to that required as a compensating balance or that which minimizes total cost d Use of checks and drafts in disbursing funds RPCPA 0595 * A company’s management is concerned about the large bank overdraft, which it wishes to reduce over the budget period of one year Which one of the four items below will not necessarily result in a lower bank overdraft? (E) a Reducing bad debts b Taking longer credit from suppliers without any loss of discounts c Reducing wastage and loss through damage of regularly-used stock items d Reducing unit costs of production RPCPA 0592 25 Methods of accelerating cash collections include all of the following except (E) A Decentralized collections C Compensating balances B Electronic funds transfers D Lockbox systems CMA 0689 1-12 RPCPA, AICPA, CMA & CIA EXAMINATION QUESTIONS 26 A typical firm doing business nationally cannot expect to accelerate its cash inflow by A Establishing multiple collection centers throughout the country B Employing a lockbox arrangement C Initiating controls to accelerate the deposit and collection of large checks CMA 1283 1-23 D Maintaining compensating balances rather than paying cash for bank services Baumol’s Model of Cash Balances Baumol's model of cash balances states that: Q = [(2 x T x C./i]^0.5 where T = annual cash disbursement, C = cost per sale of T-bills, i = interest rate What is Q? A The number of times per annum bill should be sold B The average holding of bills C The amount of T-bills that should be sold at any one time D The minimum holding of cash B&M The Baumol model determines the optimal cash balance by: A Balancing total costs against opportunity costs B Minimizing total costs of holding cash against trading securities costs C Balancing trading securities costs against total costs D Minimizing total costs less trading costs B&M 10 Concerning the Baumol model, which of the following is not correct (all other things equal)? A The optimum cash balance is higher at higher interest rates B The optimum cash balance is higher at higher fixed order costs C The optimum cash balance is higher at higher total cash requirement D All of the above are correct B&M 11 The Baumol cash balance model is limited by: A Assuming the cash flows are variable across the period B A smooth disbursement rate and now cash inflows over the period C Having a safety stock set to zero D Both B and C B&M Cash Conversion Cycle 27 An enterprise plans to produce a new product, which will typically be sold to other firms on credit The cash conversion cycle resulting from this new product can be measured as the length of time from A Cash purchases of raw materials to the collection of accounts receivable Page of 39 MANAGEMENT ADVISORY SERVICES WORKING CAPITAL FINANCE B Cash purchases of raw materials to the time the final product is completed C Cash purchases of raw materials to the sale of the product D When the product is completed to the sale of the product CIA 1193 IV-52 C Risk-profitability trade-off considerations D Flotation cost considerations CMA 0684 1-5 32 28 Helena Furnishings wants to sharply reduce its cash conversion cycle Which of the following steps would reduce its cash conversion cycle? (E) a The company increases its average inventory without increasing its sales b The company reduces its DSO c The company starts paying its bills sooner, which reduces its average accounts payable without reducing its sales d Statements a and b are correct Brigham Ignoring cost and other effects on the firm, which of the following measures would tend to reduce the cash conversion cycle? (M) a Maintain the level of receivables as sales decrease b Buy more raw materials to take advantage of price breaks c Take discounts when offered d Forgo discounts that are currently being taken e Offer a longer deferral period to customers Brigham 29 Which of the following actions are likely to reduce the length of a company’s cash conversion cycle? (M) a Adopting a just-in-time inventory system that reduces the inventory conversion period b Reducing the average days sales outstanding (DSO) on its accounts receivable c Reducing the amount of time the company takes to pay its suppliers d Statements a and b are correct Brigham Optimal Mix Between Cash and Short-term Investments 30 When managing cash and short-term investments, a corporate treasurer is primarily concerned with (E) A Maximizing rate of return B Minimizing taxes C Investing in Treasury bonds since they have no default risk D Liquidity and safety CMA 1295 1-12 31 When a company is evaluating whether the ratio of cash and marketable securities to total assets should be high or low, its decision will be based upon A Financial leverage considerations B Operating leverage considerations RPCPA, AICPA, CMA & CIA EXAMINATION QUESTIONS Some managers express the opinion that “cash management problems are nothing more than inventory problems.” They then proceed to use cash management module, such as the EOQ model to determine the a Credit and collection policies b Marketable securities level c Proper relationship between current assets and current liabilities d Proper blend of marketable securities and cash CMA 1286 1-33 33 Determining the amount and timing of conversions of marketable securities to cash is a critical element of a financial manager's performance In terms of the rate of return forgone on converted securities and the cost of such transactions, the optimal amount of cash to be raised by selling securities is A Inversely related to the rate of return forgone and directly related to the cost of the transaction B Directly related to the rate of return forgone and directly related to the cost of the transaction C Directly related to the rate of return forgone and inversely related to the cost of the transaction D Inversely related to the rate of return forgone and inversely related to the cost of the transaction CIA 0593 IV-52 34 The economic order quantity (EOQ) formula can be adapted in order for a firm to determine the optimal mix between cash and marketable securities The EOQ model assumes all of the following except a The cost of a transaction is independent of the dollar amount of the transaction and interest rates are constant over the short run b An opportunity cost is associated with holding cash, beginning with the first dollar c The total demand for cash is known with certainty d Cash flow requirements are random CMA 0689 1-15 Comprehensive * The following are desirable in cash management except: (E) a Cash is collected at the earliest time possible b Most sales are on cash basis and receivables are aged “current” c Post-dated checks are not deposited on time upon maturity Page of 39 MANAGEMENT ADVISORY SERVICES d All sales are properly receipted and promptly deposited intact * WORKING CAPITAL FINANCE RPCPA 0593 In cash management, which of the following statements is false? (M) a Capital costs, delinquency costs, and default costs are costs associated with cash management b Short costs, long costs, and procurement costs are costs associated with optimal cash balance model approach c Obtaining financing services and controlling cash flow are some of the major functions of cash management d Funds sourcing and custodianship must be done at the lowest possible cost, where excess funds must be invested for a return that is best in the market RPCPA 1090 MARKETABLE SECURITIES Criteria * The criteria that should be considered in investing surplus cash (E) a Safety of the company’s funds c Yield on the principal b Liquidity of the principal d All of the above RPCPA 0588 35 Which of the following are criteria for selection among securities available for a marketable securities portfolio? (M) A Default risk, taxability, and relative yields B Planning, major investment decisions, and interaction with capital markets C The operating, payment, and cash conversion cycles CIA 0589 IV-52 D Inventory conversion, receivables conversion, and payable deferral periods Which of the following is not a situation that might lead a firm to hold marketable securities? (M) a The firm has purchased a fixed asset that will require a large write-off of depreciable expense b The firm must meet a known financial commitment, such as financing an ongoing construction project c The firm must finance seasonal operations d The firm has just sold long-term securities and has not yet invested the proceeds in earning assets Brigham Money Market Instruments The following are money market instruments except: A T-bills C Commercial paper B Federal agency discount notes D Preferred stocks B&M The three money market securities with the greatest volume of business are: A Treasury bills, commercial paper, repurchase agreements B Negotiable CDs, federal agency discount notes, T-bills C Commercial paper, bankers' acceptances, tax-exempt municipal notes D Federal agency discount notes, repurchase agreements, medium-term notes B&M Marketable Securities Portfolio Which of the following statement completions is most correct? If the yield curve is upward sloping, then a firm’s marketable securities portfolio, assumed to be held for liquidity purposes, should be (M) a Weighted toward long-term securities because they pay higher rates b Weighted toward short-term securities because they pay higher rates c Weighted toward U S Treasury securities to avoid interest rate risk d Weighted toward short-term securities to avoid interest rate risk e Balanced between long- and short-term securities to minimize the effects of either an upward or a downward trend in interest rates Brigham RPCPA, AICPA, CMA & CIA EXAMINATION QUESTIONS Page of 39 MANAGEMENT ADVISORY SERVICES WORKING CAPITAL FINANCE Treasury Bills 36 Which security is most often held as a substitute for cash? (E) A Treasury bills C Gold B Common stock D Aaa corporate bonds CMA 1289 1-14 For which of the following investments is there a very active secondary market? A Medium-term notes C US Treasury bills B Commercial paper D Repurchase agreements B&M 37 The marketable securities with the least amount of default risk are A Federal government agency securities C Repurchase agreements B U.S Treasury securities D Commercial paper CMA 0691 1-11 38 Of the following four investments, a commercial paper b corporate bonds c Treasury bills is considered to be the safest (E) d Treasury bonds e U S Agency issues 39 Which of the following financial instruments can be traded in international money markets? A Mortgages C U.S Treasury bills B Preferred shares D U.S Treasury bonds CIA 1195 IV-65 Repurchase Agreement 15 A repurchase agreement occurs when: A A company agrees to buy back its commercial paper before maturity B A bank depositor agrees, in advance, to re-invest money in a negotiable certificate of deposit C An investor buys part of a government security dealer's inventory and simultaneously agrees to sell it back D The federal government agrees to buy T-bills B&M Negotiable Certificates of Deposit 40 Which one of the following is not a characteristic of a negotiable certificate of deposit? Negotiable certificates of deposit (D) A Have a secondary market for investors B Are regulated by the Federal Reserve System C Are usually sold in denominations of a minimum of $100,000 CMA 0691 1-12 D Have yields considerably greater than bankers' acceptances and commercial paper RPCPA, AICPA, CMA & CIA EXAMINATION QUESTIONS 10 Negotiable CDs are issued by: A US Government B Federal agencies C Banks D Corporations Commercial Paper 41 The best example of a marketable security with minimal risk would be (D) A Municipal bonds B The common stock of a Aaa rated company C The commercial paper of a Aaa rated company D Stock options of a Aaa rated company EuroCommercial Paper 14 Commercial papers sold in the international markets are called: A Negotiable commercial papers C MTNs B EuroCommercial papers D None of the above B&M CMA 0688 1-15 B&M Floating Rate Preferred Stock 16 Floating-rate preferred stock offers competitive rates of return with traditional money-market instruments but: A Is not rated by Moody's or Standard & Poor's B Still provides the corporate investor with the tax exclusion on dividend income C Has a fixed rate of dividend income D Offers a highly competitive trading market B&M 17 A tax-paying corporation would prefer to invest short-term money in: A Preferred stock C Common stock B Floating-rate preferred stock D Long-term bonds B&M 21 Even though the dividend rate on a floating-rate preferred stock is floating to keep in line with interest rates, the instrument still suffers from risk such as: A A thin market causing potential principal risk and liquidity concerns B The risk of downgrades from the narrow range of issuers C The impact of tax law changes, which may reduce the after-tax value of the instrument D All of the above B&M Page 10 of 39 14 REQUIRED: The false statement about working capital DISCUSSION: (D) Financing permanent inventory build-up which is essentially a long-term investment, with longterm debt is a moderate or conservative working capital policy An aggressive policy uses short-term relatively lowcost debt to finance the inventory buildup It focuses on high profitability potential, despite high risk and low liquidity An aggressive policy reduces the current ratio and accepts a higher risk of short-term lack of liquidity Financing inventory with long-term debt increases the current ratio and accepts higher borrowing costs in exchange for greater liquidity and lower risk Answer (A) is incorrect because current liabilities, e.g., trade credit is a major source of funds for small firms Answer (B) is incorrect because liquid investments tend to have lower returns Answer (C) is incorrect because matching of asset and liability maturities is a moderate policy that minimizes risk The expectation is that cash flows from the assets will be available to meet obligations for the liabilities 15 REQUIRED: The primary concern when managing cash and short-term investments DISCUSSION: (E) Cash and short-term investments are crucial for a firm’s continuing success Sufficient liquidity must be available to meet payments as they come due At the same time, liquid assets are subject to significant control risk Therefore, liquidity and safety are the primary concerns of the treasurer when dealing with a highly liquid assets Cash and short-term investments are held because of their ability to facilitate routine operations of the company These assets are not held for purposes of achieving investment returns Answer (A) is incorrect because most companies are not in business to earn high returns on liquid assets (i.e., they are held to facilitate operations) Answer (B) is incorrect because the holding of cash and cash-like assets is not a major factor in controlling taxes Answer (C) is incorrect because investments in Treasury bonds not have sufficient liquidity to serve as short-term assets Answer (D) is incorrect because common stocks not have sufficient liquidity to serve as short-term cash resources 16 A very efficient cash management system could allow a firm to operate with positive net float where the firm has a negative checkbook balance at most times but still does not bounce its checks The other statements are false A good cash management system maximizes disbursement float and minimizes collections float A well-designed lockbox system minimizes collections float which would increase a firm’s net float Increases in interest rates raise the opportunity cost of idle cash A firm prefers to write checks, maximizing its disbursement float and increasing its net float 17 Answer (D) is correct John Maynard Keynes, founder of Keynesian economics, concluded that there were three major motives for holding cash: for transactional purposes as a medium of exchange, precautionary purposes, and speculative purposes (but only during deflationary periods) Answer (A) is incorrect because the three major motives for holding cash, according to Keynesian economics, are for transactional, precautionary, and speculative purposes Answer (B) is incorrect because the three major motives for holding cash, according to Keynesian economics, are for transactional, precautionary, and speculative purposes Answer (C) is incorrect because the three major motives for holding cash, according to Keynesian economics, are for transactional, precautionary, and speculative purposes 18 Answer (D) is correct Companies hold cash to facilitate routine transactions, to compensate banks for providing loans and services, to guard against unforeseen fluctuations in cash flows, and to take advantage of opportunities for bargain purchases However, firms are more likely to hold trading securities or rely on easy access to credit to take advantage of bargains or to guard against unforeseen fluctuations in cash flows Answer (A) is incorrect because a transactions balance is associated with routine payments and collections Answer (B) is incorrect because a compensating balance is the minimum amount on deposit at a bank to compensate the bank for providing loans and other services Answer (C) is incorrect because a precautionary balance consists of reserves for unforeseen fluctuations in cash flows 19 Answer (D) is correct A company will hold cash and marketable securities to facilitate business transactions because cash is a primary medium of exchange Cash and near-cash items are also held to meet future needs, to satisfy compensating balance requirements imposed by lenders, and to provide a precautionary balance for security purposes Cash is usually not held in an attempt to earn maximum returns on investment because cash and marketable securities are not usually the highest-paying investments Answer (A) is incorrect because cash is held to satisfy compensating balance requirements Answer (B) is incorrect because cash is held to facilitate transactions Answer (C) is incorrect because cash is held to meet future needs 20 Net float = Disbursements float - Collections float; therefore the larger the disbursements float and the lower the collections float the better the cash management system A lockbox is used to speed cash collections If a firm’s outflows come due early in the month rather than uniformly this will necessitate a large line of credit 21 Answer (D) is correct A lock-box system accelerates the inflow of funds A company maintains mail boxes, often in numerous locations around the country, to which customers send payments A bank checks these mailboxes several times a day, and funds received are immediately deposited to the company's account without first being processed by the company's accounting system, thereby hastening availability of the funds Answer (A) is incorrect because a lock-box system is not related to compensating balances; a compensating balance may be required by a covenant in a loan agreement that requires a company to maintain a specified balance during the term of the loan Answer (B) is incorrect because a lock-box system is a process by which payments are sent to a bank's mailbox, which is checked during normal post office hours Answer (C) is incorrect because the use of a lock-box system entails sending checks through the mail to a post office box Thus, it does not reduce the risk of losing checks in the mail 22 REQUIRED: The working capital technique that delays the outflow of cash DISCUSSION: A draft is a three-party instrument in which one person (the drawer) orders a second person (the drawee) to pay money to a third person (the payee) A check is the most common form of draft It is an instrument payable on demand in which the drawee is a bank Consequently, a draft can be used to delay the outflow of cash A draft can be dated on the due date of an invoice and will not be processed by the drawee until that date, thereby eliminating the necessity of writing a check earlier than the due date or using an EFT Thus, the outflow is delayed until the check clears the drawee bank Answer (A) is incorrect because factoring is the sale of receivables and therefore concerns cash inflows, not outflows Answer (B) is incorrect because a lockbox system is a means of accelerating cash inflows Answer (D) is incorrect because an electronic funds transfer results in an immediate deduction from the payor’s bank account, thereby eliminating float 23 Answer (B) is correct Payment by draft, a three-party instrument in which the drawer orders the drawee to pay money to the payee, is a means of slowing cash outflows A check is the most common type of draft Check float arises from the delay between an expenditure and the clearing of the check through the banking system Answer (A) is incorrect because concentration banking, a lockbox system, and the use of a local post office box are techniques used to accelerate cash receipts Answer (C) is incorrect because EDI is the communication of electronic documents directly from a computer in one entity to a computer in another entity Thus, EDI expedites cash payments The payee receives the money almost instantaneously Answer (D) is incorrect because concentration banking, a lockbox system, and the use of a local post office box are techniques used to accelerate cash receipts 24 REQUIRED: The definition of an automated clearing house (ACH) electronic transfer DISCUSSION: (A) An ACH electronic funds transfer (EFT) is an electronic payment to a company’s account of a concentration bank A concentration bank is a large bank to which a company transfers funds from local depository banks These local banks operate the company’s lockboxes and thus serve as collection points The transfer of funds to the concentration bank allows the company to take advantage of economies of scale in cash management The use of an ACH facilitates concentration banking ACHs are electronic networks operated by the Federal Reserve (except for the New York regional ACH association) that guarantee 1-day clearing Answer (B) is incorrect because a check is not involved in an EFT Answer (C) is incorrect because an ACH transfer involves the actual transfer of funds electronically; it is not just a computer generated document Answer (D) is incorrect because an EFT is not a check-like instrument 25 Answer (C) is correct Various methods of accelerating cash collections include decentralized collection outposts (normally one in each Federal Reserve District), electronic funds transfers, centralized banking for all company branches to avoid having to maintain minimum balances in several locations, and lockbox systems A compensating balance is a minimum average or absolute amount that must be maintained in a bank account Hence, it is not a means of accelerating cash collections This requirement means that less cash is available to the depositor Answer (A) is incorrect because it is a common method of accelerating cash collections Answer (B) is incorrect because it is a common method of accelerating cash collections Answer (D) is incorrect because it is a common method of accelerating cash collections 26 Answer (D) is correct Compensating balances are either (1) an absolute minimum balance or (2) a minimum average balance that bank customers must keep at the bank These are generally required by the bank to compensate for the cost of services rendered Maintaining compensating balances will not accelerate a company's cash inflows because less cash will be available even though the amount of cash coming in remains unchanged Answer (A) is incorrect because multiple collection centers throughout the country will reduce the time required to receive cash in the mail For example, California customers of a New York firm would make payment to a West Coast center Thus, the company would receive the cash two or three days sooner Answer (B) is incorrect because direct deposit by customers into a lock-box also speeds cash into company accounts Answer (C) is incorrect because special handling of large checks is a cost-effective way to deposit large amounts 27 Answer (A) is correct Cash conversion is the process of converting cash to products and back to cash The length of the cycle is from the cash outflow to the cash inflow Thus, the cash conversion cycle, in the case of production, begins with the cash purchase of raw materials and ends with the collection of accounts receivable Answer (B) is incorrect because the cash inflows have not been received Answer (C) is incorrect because the cash inflows have not been received Answer (D) is incorrect because the cash inflows have not been received 28 Statement a is false If inventory increases, and sales not, more cash is being “tied up” in inventory so the cash conversion cycle is increased, not reduced Statement b is true If the company reduces its DSO, it is collecting its accounts receivables more efficiently, so it reduces the cash conversion cycle Statement c is false If the company pays its bills sooner, it uses its cash to pay off accounts payable, which increase its cash conversion cycle 29 Statements a and b are correct; therefore, statement d is the appropriate choice Delaying payments to suppliers increases the length of the cash conversion cycle 30 Answer (D) is correct Cash and short-term investments are crucial to a firm's continuing success Sufficient liquidity must be available to meet payments as they come due At the same time, liquid assets are subject to significant control risk Therefore, liquidity and safety are the primary concerns of the treasurer when dealing with highly liquid assets Cash and short-term investments are held because of their ability to facilitate routine operations of the company These assets are not held for purposes of achieving investment returns Answer (A) is incorrect because most companies are not in business to earn high returns on liquid assets (i.e., they are held to facilitate operations) Answer (B) is incorrect because the holding of cash and cash-like assets is not a major factor in controlling taxes Answer (C) is incorrect because investments in Treasury bonds not have sufficient liquidity to serve as short-term assets 31 Answer (C) is correct The trade-off between risk and return must be considered because liquid assets are usually less profitable than less-liquid alternatives However, a greater liquidity means less risk of being unable to meet obligations when they are due Answer (A) is incorrect because financial leverage concerns the extent to which debt financing is used Answer (B) is incorrect because operating leverage concerns the proportion of fixed operating costs Answer (D) is incorrect because the costs of issuing securities relate to capital structure finance 32 REQUIRED: The true statement about cash management models DISCUSSION: (D) Because cash and inventory are both non-earning assets, in principle they may be treated similarly The alternative to holding cash, however, is to hold marketable securities that earn interest or dividends Thus, a cash management model determine how much of a firm’s liquidity should be held as cash and how much in the form of marketable securities Answer (A) is incorrect because credit and collection policies concern receivables and are not influenced by an EOQ model for inventory management Answer (B) is incorrect because the level of marketable securities is in part determined by cash needs, so (D) is a better answer Answer (C) is incorrect because the relationship between current assets and current liabilities concerns many factors other than cash management 33 Answer (A) is correct The optimal amount of cash to be raised by selling securities is calculated by a formula similar to that used to determine the economic order quantity for inventory C+ = 2(F)(T) k If: C+ = Cash to be raised T = Total cash needed for the period F = Cost of making a securities trade k = Opportunity cost of holding cash The optimal amount of cash to be raised by selling securities is inversely related to the rate of return forgone (opportunity cost) and directly related to the cost of the transaction Answer (B) is incorrect because a high (low) opportunity cost results in a lower (higher) optimal cash balance, whereas high (low) transaction costs result in a higher (lower) optimal cash balance Answer (C) is incorrect because a high (low) opportunity cost results in a lower (higher) optimal cash balance, whereas high (low) transaction costs result in a higher (lower) optimal cash balance Answer (D) is incorrect because a high (low) opportunity cost results in a lower (higher) optimal cash balance, whereas high (low) transaction costs result in a higher (lower) optimal cash balance 34 REQUIRED: The assumption not made in the EOQ model DISCUSSION: (D) The EOQ formula is a deterministic model that requires a known demand for inventory, or in this case, the amount of cash needed Thus, the cash flow requirements cannot be random The model also assumes a given carrying (interest) cost and a flat transaction cost for converting marketable securities to cash, regardless of the amount withdrawn Answer (A) is incorrect because the EOQ model assumes that the cost of a transaction is independent of the dollar amount of the transaction and interest rates are constant over the short-run Answer (B) is incorrect because the EOQ model assumes an opportunity cost is associated with holding cash, beginning with the first dollar Answer (C) is incorrect because the EOQ model assumes that the total demand for cash is known with certainty 35 Answer (A) is correct Risk and yield are fundamental concerns, and taxation is a pervasive issue that affects yield Default risk is the risk that a borrower will be unable to make interest payments or principal repayments on debt (e.g., bonds purchased from a financially troubled company have high default risk) Taxability refers to the tax treatment of any income or loss from a security Relative yields are the rates of return of each security in comparison with other potential investments Answer (B) is incorrect because these matters are a financial manager's overall responsibility, not securities selection criteria Answer (C) is incorrect because these cycles are important in working capital management The operating and payment cycles are components of the cash conversion cycle This cycle lasts from the time cash is paid for resources to the time cash is received in payment for goods sold Answer (D) is incorrect because these are factors in the determination of cash flow The inventory conversion cycle is the time between acquisition of resources and sale of finished goods The receivables conversion cycle lasts from the date of sale of finished goods to the date of cash collection A payable deferral period is the lapse of time between purchase of resources and the date they are paid for Thus, the cash conversion cycle does not begin until the end of the payable deferral period 36 Answer (A) is correct A Treasury bill is a short-term U.S government obligation that is sold at a discount from its face value A Treasury bill is highly liquid and nearly risk-free, and it is often held as a substitute for cash Answer (B) is incorrect because common stock lacks the liquidity necessary to be a cash substitute Common stock can also be a risky investment Answer (C) is incorrect because gold lacks the liquidity necessary to be a cash substitute Gold can also be a risky investment Answer (D) is incorrect because Aaa corporate bonds lack the liquidity necessary to be a cash substitute Aaa corporate bonds can also be risky investments 37 Answer (B) is correct The marketable securities with the lowest default risk are those issued by the federal government because they are backed by the full faith and credit of the U.S Agency securities are issued by agencies and corporations created by the federal government, such as the Federal Housing Administration They are backed by a secondary promise from the government Answer (A) is incorrect because securities issued by a federal agency are first backed by that agency and secondarily by the U.S government Answer (C) is incorrect because repurchase agreements could become worthless if the organization agreeing to make the repurchase goes bankrupt Answer (D) is incorrect because commercial paper is unsecured 38 Only Treasury issues are insured by the U S government; the shorter-term the instrument, the safer the instrument 39 Answer (C) is correct Funds are borrowed or lent for short periods (less than one year) in money markets Examples of instruments traded in money markets are U.S Treasury bills, bankers' acceptances, commercial paper, negotiable certificates of deposit, money market mutual funds, Eurodollar market time deposits, and consumer credit loans Capital markets trade shares and long-term debt Answer (A) is incorrect because mortgages are long-term capital market securities Answer (B) is incorrect because preferred shares are long-term capital market securities Answer (D) is incorrect because U.S Treasury bonds are long-term capital market securities 40 Answer (D) is correct A certificate of deposit (CD) is a form of savings deposit that cannot be withdrawn before maturity without incurring a high penalty A negotiable CD can be traded CDs usually have a fairly high rate of return compared with other savings instruments because they are for fixed, usually long-term periods However, their yield is less than that of commercial paper and bankers' acceptances because they are less risky Answer (A) is incorrect because negotiable CDs have a secondary market (i.e., they are negotiable) Answer (B) is incorrect because negotiable CDs are regulated Answer (C) is incorrect because negotiable CDs are typically issued in a denomination of $100,000 41 Answer (C) is correct Of the choices given, the commercial paper of a top-rated (most creditworthy) company has the least risk Commercial paper is preferable to stock or stock options because the latter represent only a residual equity in a corporation Commercial paper is debt and thus has priority over stockholders' claims Also, commercial paper is a very short-term investment The maximum maturity allowed without SEC registration is 270 days However, it can be sold only to sophisticated investors without registration Answer (A) is incorrect because municipal bonds are rarely considered marketable securities in that they constitute long-term debt Answer (B) is incorrect because common stock does not have as high a priority in company assets as commercial paper or other debt Answer (D) is incorrect because common stock does not have as high a priority in company assets as commercial paper or other debt 42 Answer (C) is correct A small firm with surplus cash should invest for the highest return and lowest risk The ability to convert the investment into cash without a loss of principal is also important Money market mutual funds invest in money market certificates such as treasury bills, negotiable CDs, and commercial paper Because of diversification, these mutual funds are superior to any single instrument Answer (A) is incorrect because a small firm may not have enough surplus cash to invest in commercial paper, which usually consists of secured or unsecured promissory notes of large corporations Answer (B) is incorrect because the transactions cost of bankers' acceptances is high A banker's acceptance is a unique credit instrument used to finance both domestic and international "self-liquidating" transactions It is usually initiated by a bank's irrevocable letter of credit on behalf of the bank's customer, on which the company doing business with the bank's customer draws a time draft The company discounts the time draft with the company's local bank and receives immediate payment The local bank forwards the time draft to the bank customer for payment Answer (D) is incorrect because an increase in interest rates could cause a substantial loss in principal 43 Answer (B) is correct Eurobonds are, by definition, always sold in some country other than the one in whose currency the bond issue is denominated Their advantage is that they are customarily less stringently regulated than most other bonds Hence, transaction costs are lower Answer (C) is incorrect because foreign bonds are denominated in the currency of the country in which they are sold (A) is incorrect because Eurobonds are not always denominated in Eurodollars, which are U.S dollars deposited outside the U.S Answer (D) is incorrect because Eurobonds are usually issued not as registered bonds but as bearer bonds, so names and nationalities of the investors are not recorded 44 Answer (C) is correct Eurodollars are U.S dollars on deposit in a foreign bank These deposits are created when a check is drawn on a dollar deposit in a U.S bank and then deposited in a bank outside the U.S This amount is then available for lending by the foreign bank to its customers However, the depositors still hold claims denominated in dollars Because Eurodollars are outside the direct control of the U.S monetary authorities, U.S banking regulations with respect to reserves, insurance, interest ceilings, etc., not apply The absence of these costs means that Eurodollar deposit rates tend to be higher, not lower, than domestic U.S rates on equivalent instruments Answer (A) is incorrect because these deposits are outside the control of U.S authorities because they are in banks outside the U.S Answer (B) is incorrect because these deposits are outside the control of U.S authorities because they are in banks outside the U.S Answer (D) is incorrect because interest rates on these deposits, and loans made thereon, are tied to the LIBOR, which is the rate paid on deposits of other large banks by the largest London banks with the highest credit standing 45 Answer (D) is correct Eurodollars are U.S dollars deposited in banks outside the U.S Because it is outside the direct control of the U.S monetary authorities, the Eurodollar market has lower costs For example, U.S reserve requirements and FDIC premium payments not apply in this market A lower cost market can offer depositors higher interest rates Answer (A) is incorrect because Eurodollar borrowers tend to pay lower, not higher, rates Borrowers and depositors can both receive more favorable rates because, with its lower costs, the Eurodollar market can offer smaller spreads between borrowing and lending rates Answer (B) is incorrect because U.S dollars are on deposit in both cases Answer (C) is incorrect because Eurodollar deposits tend to be for larger, not smaller, amounts Furthermore, smaller deposits tend to earn lower, not higher, rates than larger deposits 46 Answer (D) is correct Marketable securities are near-cash items used primarily for short-term investment Examples include U.S Treasury bills, Eurodollars, commercial paper, money-market mutual funds with portfolios of short-term securities, bankers' acceptances, floating-rate preferred stock, and negotiable CDs of U.S banks A convertible bond is not a short-term investment because its maturity date is usually more than one year in the future and its price can be influenced substantially by changes in interest rates or by changes in the investee's stock price Answer (A) is incorrect because U.S Treasury bills are short-term marketable securities Answer (B) is incorrect because Eurodollars are short-term marketable securities Answer (C) is incorrect because commercial paper is a short-term marketable security 47 REQUIRED: The definition of “short-selling” DISCUSSION: (B) Short-selling is accomplished by borrowing securities from a broker and selling those securities At a later time, the loan is repaid by buying securities on the open market and returning them to th broker The seller speculates that the stock’s market price will decline Answer (A) is incorrect because margin trading involves buying securities bu borrowing money from a broker Answer (C) is incorrect because the investor does not own the shares sold in a short sale Answer (D) is incorrect because the short seller is betting that the stock will decrease in price 48 Answer (C) is correct The day's sales outstanding (days of receivables) may be stated as the accounts receivable balance divided by average credit sales per day or as days in the year divided by the receivables turnover It is the average time required to convert the enterprise's receivables into cash Thus, it is also called the receivables collection period Answer (A) is incorrect because the inventory conversion period (days of inventory) is the average time required to convert materials into finished goods and then to sell them This process typically occurs before the receivables collection period, and the amount of time in one period does not necessarily bear any relationship to the other Answer (B) is incorrect because the cash conversion cycle equals the inventory conversion period, plus the receivables collection period, minus the payables deferral period (average time between resource purchases and payment of cash for them) It estimates the time between when the enterprise makes payments and when it receives cash inflows Answer (D) is incorrect because the inventory divided by the sales per day is the inventory conversion period (days of inventory) 49 REQUIRED: The meaning of a firm’s average collection period DISCUSSION: (A) The average collection period measures the number of days between the date of sale and the date of collection It should be related to a firm’s credit terms For example, a firm that allows terms of 2/15, net 30 should have an average collection period of somewhere between 15 and 30 days Answer (B) is incorrect because it describes the concept of float Answer (C) is incorrect because the average collection period includes the total time before a payment is received Including the periods both before and after the end of the normal credit period Answer (D) is incorrect because it describes the normal credit period 50 REQUIRED: The item measured by an aging of accounts receivable DISCUSSION: (D) The purpose of an aging of receivables is to classify receivables by due date Those that are current (not past due) are listed in one column, those less than 30 days past due in another column, etc The amount in each category can then be multiplied by an estimated bad debt percentage that is based on a company’s credit experience and other factors The theory is that the oldest receivables are the least likely to be collectible Aging the receivables and estimating the uncollectible amounts is one method of arriving at the appropriate balance sheet valuation of the accounts receivable account Answer (A) is incorrect because an aging schedule is used for receivables, not liabilities Answer (B) is incorrect because an aging schedule concerns specific accounts, not averages Answer (C) is incorrect because an aging schedule focuses on uncollectible receivables 51 Answer (B) is correct A quantity discount is an attempt to increase sales by reducing the unit price on bulk purchases It concerns only the price term of an agreement, not the credit term, and thus is unrelated to credit and collection policy Answer (A) is incorrect because the quality of accounts is important to credit policy since it is inversely related to both sales and bad debts Answer (C) is incorrect because offering a cash discount improves cash flow and reduces receivables and the cost of extending credit Answer (D) is incorrect because the level of collection expenditures must be considered when implementing a collection policy The marginal cost of a credit and collection policy should not exceed its revenue 52 Answer (D) is correct Increasing the quality of the accounts rejected means that fewer sales will be made The company is therefore not trying to maximize its sales or increase its bad debt losses The objective is to reduce bad debt losses and thereby maximize profits Answer (A) is incorrect because tightening credit will reduce sales and bad debt losses Answer (B) is incorrect because tightening credit will reduce sales and bad debt losses Answer (C) is incorrect because, most likely, higher quality accounts will mean a shorter average collection period 53 REQUIRED: The calculation of the average gross receivable balance DISCUSSION: A firm’s average gross receivables balance can be calculated by multiplying average daily sales by the average collection period (days’ sales outstanding) Alternatively, annual credit sales can be divided by the accounts receivable turnover (net credit sales ÷ average accounts receivable) to obtain the average balance in receivables Answers (A) and (B) are incorrect because Alternative cannot be correct Neither of the multiplicands is a dollar figure, sot the product could not be the dollar balance of receivables Answer (D) is incorrect because Alternative III cannot be correct It contains average gross receivables, the amount being calculated 54 Answer (C) is correct A high receivables turnover is indicative of short credit terms and possibly the use of a lockbox system to speed up cash flows Answer (A) is incorrect because a lenient credit policy would lead to a low turnover Answer (B) is incorrect because small discounts although helpful, would be unlikely to lead to an unusually high turnover Small discounts would not be as helpful as a lockbox system and short credit terms Answer (D) is incorrect because a high turnover would be indicative of an efficient credit and collection department 55 Answer (B) is correct A decline in the receivable turnover indicates that either sales are declining relative to receivables, or receivables are increasing relative to sales Thus, the firm is collecting credit sales more slowly than before Answer (A) is incorrect because sales are being collected more slowly when the turnover declines Answer (C) is incorrect because an increase in sales will increase the turnover ratio Answer (D) is incorrect because inventories have no impact on the receivable turnover ratio 56 Answer (D) is correct The accounts receivable turnover ratio equals net credit sales divided by average receivables Hence, it will decrease if a company lengthens the credit period or the discount period because the denominator will increase as receivables are held for longer times Answer (A) is incorrect because write-offs not reduce net receivables (gross receivables - the allowance) and will not affect the receivables balance and therefore the turnover ratio if an allowance system is used Answer (B) is incorrect because a decline in sales near the end of the period signifies fewer credit sales and receivables, and the effect of reducing the numerator and denominator by equal amounts is to increase the ratio if the fraction is greater than 1.0 Answer (C) is incorrect because an increase in cash sales with no diminution of credit sales will not affect receivables 57 REQUIRED: The reason for offering credit terms of 2.10, net 30 DISCUSSION: (D) Because these terms involve an annual interest cost of over 36%, a company would not offer them unless it desperately needed cash Also, credit terms are typically somewhat standardized within an industry Thus if most companies in the industry offer similar terms, a firm will likely be forced to match the competition or lose market share Answer (A) is incorrect because, if the company does not need cash, it would not offer cash discounts, regardless of its cost of capital, unless required to match competition Answer (B) is incorrect because the ability to borrow at a lower rate is a reason for not offering cash discounts Answer (C) is incorrect because the relationship between the cost of capital and the prime rate may not be relevant if the firm cannot borrow at the prime rate 58 REQUIRED: The true statement about extending credit for a period longer than the purchase’s operating cycle DISCUSSION: (B) The normal operating cycle is defined as the period from the acquisition of inventory to the collection of the account receivable If trade credit is for a period longer than the normal operating cycle, the seller must therefore be financing more than just the purchase of inventory Answer (A) is incorrect because a seller that extends long-term credit will have a higher level of receivables than a firm with a shorter credit period Answer (C) is incorrect because the seller is not guaranteed that a purchaser will resell the merchandise Answer (D) is incorrect because offering a discount may accelerate payment 59 Answer (C) is correct If the cause of increased sales is an increase in the cash discount, it can be inferred that the additional customers would pay during the discount period Thus, cash would be collected more quickly than previously and the cash conversion cycle would be shortened Answer (A) is incorrect because the operating cycle would decrease since the average time from cash disbursement to cash realization would be shorter Answer (B) is incorrect because the average collection period would decrease since the average time from cash disbursement to cash realization would be shorter Answer (D) is incorrect because more customers will take discounts 60 REQUIRED: The true statement about a change in credit policy that has resulted in greater sales and a reduction in accounts receivable DISCUSSION: (A) An increase in discounts taken accompanied by declines in receivables balances and doubtful accounts all indicate that collections on the increased sales have been accelerated Accordingly, the average collection period is a ratio calculated by dividing the number of days in a year (365) by the receivable turnover Thus, the higher the turnover, the shorter the average collection period The turnover increases when either sales (the numerator) increase or receivables (the denominator) decrease Accomplishing both higher sales and a lower receivables increases the turnover and results in a shorter collection period Answer (B) is incorrect because a decrease in the percentage discount offered provides no incentive for early payment Answer (C) is incorrect because accounts receivable turnover (sales ÷ average receivables) has increased Answer (D) is incorrect because no information is given relative to working capital elements other than receivables Both receivables and cash are elements of working capital, so an acceleration of customer payments will have no effect on working capital 61 REQUIRED: The true statement about a change in credit policy that has resulted in greater sales and a reduction in accounts receivable DISCUSSION: (B) An increase in discounts taken accompanied by declines in receivables balances and doubtful accounts all indicate that collections on the increased sales have been accelerated Accordingly, the average collection period must have declined The average collection period is a ratio calculated by dividing the number of days in a year (365) by the receivable turnover Thus, the higher the turnover, the shorter the average collection period The turnover increases when either sales (the numerator) increase, or receivables (the denominator) decrease Accomplishing both higher sales and a lower receivables increases the turnover and results in a shorter collection period Answers (A) and (C) are incorrect because no statement can be made with respect to profits without knowing costs Answer (D) is incorrect because the discount may have been increased, which has led to quicker payments Answer (E) is incorrect because the bad debt loss percentage has apparently declined Sales are up; doubtful accounts are fewer 62 REQUIRED: The factor not considered in an analysis of propose credit policies DISCUSSION: (B) All factors should be considered that differ between the two policies Factors that not differ, such as the current bad debt experience, are not relevant Ryan must estimate the expected bad debt losses under each new policy Answer (A) is incorrect because the cost of funds is an obvious element in the analysis of any investment Answer (C) is incorrect because the impact on the current customer base of extending terms to only certain customers is relevant The current customers may demand the same terms Answer (D) is incorrect because existing loan agreements may require Ryan to maintain certain ratios at stated levels Thus, Ryan’s ability to increase receivables and possible bad debt losses may be limited 63 Answer (A) is correct Relaxing the credit policy for customers will lead to increased sales because more people will be eligible for more credit As sales increase, purchase orders will increase to accommodate the higher sales levels Answer (B) is incorrect because inventory should be increased to accommodate higher sales levels Answer (C) is incorrect because safety stock is based on expected sales, which are expected to rise Answer (D) is incorrect because a just-in-time system is not used when a company orders inventory once a quarter 64 Answer (A) is correct A factor purchases a company's accounts receivable and assumes the risk of collection The seller receives money immediately to reinvest in new inventories The financing cost is usually high: about points or more above prime, plus a fee for collection Factoring has been traditional in the textile industry for years, and recently companies in many industries have found it an efficient means of operation A company that uses a factor can eliminate its credit department, accounts receivable staff, and bad debts These reductions in costs can more than offset the fee charged by the factor, which can often operate more efficiently than its clients because of the specialized nature of its service Answer (B) is incorrect because factoring is a source of short-term funds through sale of receivables Answer (C) is incorrect because factoring is a source of short-term funds through sale of receivables Answer (D) is incorrect because factoring is a source of short-term funds through sale of receivables 65 Answer (B) is correct Pledging accounts receivable is an arrangement in which receivables are used as security for a loan But if the receivables are not paid, the secured party still may proceed against the borrowers Lenders in these circumstances choose the collateral with care, retain the right of recourse against the borrower in case of default, and usually lend some amount less than the face value of the receivables Answer (A) is incorrect because factoring is the outright sale of receivables for cash at a discount Answer (C) is incorrect because receivables may be sold with or without recourse Answer (D) is incorrect because hedging is the process of protecting oneself against loss because of future price changes For example, a party that has agreed to deliver a commodity at a certain date may enter into a hedging contract to buy an equal quantity on the same date Any loss on one transaction should then be offset by the gain on the other 66 Answer (A) is correct An increase in the proportion of short-term financing will not affect a company's degree of leverage, but risk is increased because of the need for frequent refinancing Because the debtor company will be forced to meet principal and interest payments quickly, perhaps before expected funds from a new project, the danger of default is increased Also, future interest rates are difficult to predict Answer (B) is incorrect because leverage is the use of borrowed funds to earn returns for stockholders It is irrelevant whether the borrowed funds are long- or short-term Answer (C) is incorrect because the length of a loan does not affect the amount of liquid assets Both long- and short-term loans result in liquid assets Answer (D) is incorrect because an increase in current liabilities decreases the current ratio 67 Under normal conditions the yield curve is upward sloping, thus, short-term interest rates are lower than long-term interest rates Consequently, a firm financing with short-term debt will pay less interest than a firm financing with long-term debt increasing its ROE However, a firm increases its risk by financing with short-term debt because such debt must be “rolled over” frequently, and the firm is exposed to the volatility of short-term rates The other statements are false 68 REQUIRED: The item that provides a spontaneous source of financing DISCUSSION: (A) Trade credit is a spontaneous source of financing because it arises automatically as part of a purchase transaction Because of its ease in use, trade credit is the largest source of short-term financing for may firms both large and small Answers (B) and (D) are incorrect because mortgage bonds and debentures not arise automatically as a result of a purchase transaction Answer (C) is incorrect because the use of receivables as a financing source requires an extensive factoring arrangement and often involves the creditor’s evaluation of the credit ratings of the borrower’s customers 69 Answer (D) is correct Trade credit is a spontaneous source of financing because it arises automatically as part of the purchase transaction Answer (A) is incorrect because it occurs as a result of transactions apart from purchase transactions In other words, such credit is arranged separately from the transactions to acquire the assets being financed Answer (B) is incorrect because it occurs as a result of transactions apart from purchase transactions In other words, such credit is arranged separately from the transactions to acquire the assets being financed Answer (C) is incorrect because prepaid interest is not a source of financing 70 Answer (C) is correct Trade credit is a spontaneous source of financing because it arises automatically as part of a purchase transaction Because of its ease in use, trade credit is the largest source of short-term financing for many firms both large and small Answer (A) is incorrect because installment loans are usually a longer-term source of financing and are more difficult to acquire than trade credit Answer (B) is incorrect because commercial paper is normally used only by large companies with high credit ratings Answer (D) is incorrect because mortgage bonds are a long-term source of financing 71 REQUIRED: The true statement about trade credit DISCUSSION: (C) Trade credit is a spontaneous source of financing because it arises automatically as part of a purchase transaction The terms of payment are set by the supplier, but trade credit usually requires payment within a short period of time Trade credit is an important source of credit for all businesses but especially for buyers, such as small businesses but especially for buyers, such as small businesses, that might not have access to other credit markets Like all forms of financing, trade credit is subject to the risk of buyer default Answer (A) is incorrect because trade credit is an important source of financing for small firms Answer (B) is incorrect because trade credit is ordinarily short-term source of financing Answer (D) is incorrect because the cost of trade credit depends on the credit terms and the price paid A seller with generous payment terms may charge a higher price for its merchandise 72 Answer (A) is correct The first term of the formula represents the periodic cost of the trade discount, calculated as the cost per unit of trade credit (discount %) divided by the funds made available by not taking the discount (100 discount %) The second term represents the number of times per year this cost is incurred The multiple of these terms is the approximate annual percentage cost of not taking the trade discount A precise formula would incorporate the effects of compounding when calculating the annual cost Answer (B) is incorrect because the denominator of the first term should represent the funds made available by not taking the discount (100 - discount %) Answer (C) is incorrect because the first term is the reciprocal of the correct term Answer (D) is incorrect because the second term is the reciprocal of the correct term 73 Answer (D) is correct If the discount period is longer, the days of extra credit obtained by forgoing the discount are fewer Assuming other factors are constant, the result is that the cost of trade credit, that is, the cost of not taking the discount, is greater Answer (A) is incorrect because the lower the discount percentage, the lower the opportunity cost of forgoing the discount and using the trade credit financing Answer (B) is incorrect because percentage financing cost is unaffected by the purchase price of the items Answer (C) is incorrect because percentage financing cost is unaffected by the purchase price of the items 74 REQUIRED: The true statement about cash discounts DISCUSSION: (D) Payments should be made within the discount periods if the cost of not taking discounts exceed the firm’s cost of capital For example, failing to take a discount when terms are 2/10, net 30 means that the firm is paying an effective annual interest rate exceeding 36% Thus the cost of not taking the discount is usually higher than the cost of a bank loan Answer (A) is incorrect because the cost of not taking a discount when terms are 2/10, net 30 exceeds 36% annually, which is higher than the prime rate has ever been Answer (B) is incorrect because the buyer is paying the amount of discount not taken in exchange for the extra 45 days of credit Answer (C) is incorrect because paying 2% for 20 days of credit is more expensive than paying 2% for 50 days of the same amount of credit 75 Answer (C) is correct A small retail store would not have access to major capital markets In fact, the only options available, outside of owner financing, are bank loans and a line of credit from suppliers It is this latter alternative that is most often used because it permits the store to finance inventories for 30 to 60 days without incurring interest cost A line of credit is an arrangement between a bank and a borrower in which the bank commits itself to lend up to a certain maximum amount to the borrower in a given period Answer (A) is incorrect because only large companies with excellent credit ratings have access to the commercial paper market Answer (B) is incorrect because a retail store must have instant access to its inventory to provide continuous services to customers Thus, a terminal warehouse receipt loan would not be suitable because the inventory would not be in the immediate possession of the seller Answer (D) is incorrect because a chattel mortgage is most often used for financing movable equipment It is not well-suited to financing inventory of a small retailer with high turnover because of the difficulty of identification 76 Statement b illustrates an aggressive financing policy, not a conservative one 77 Answer (D) is correct The first step is to determine the actual annual percentage interest rate for each of the four options Assuming a $100 invoice, the Fort Company discount represents interest of $1 on a loan of $99 for 20 days (30-day credit period - 10-day discount period) The annual interest rate is 18.1818% [(360/20) periods x ($1/$99)] The Riley Company discount represents an interest charge of $2 on a loan of $98; i.e., by not paying on the 15th day, the company will have the use of $98 for 45 days (60-day credit period - 15-day discount period) The number of periods in a year would be (360/45) The interest would be 16.326% ($2/$98 x periods) The Shad loan would be for $97 at a cost of $3 The loan would be for 75 days (90 - 15) Given 4.8 interest periods in a year (360/75), the annual interest rate would be 14.845% ($3/$97 x 4.8) The bank loan was quoted at 14.75% on a discount basis On a $100 note, the borrower would only receive $85.25, giving an interest rate of 17.302% ($14.75/$85.25) Thus, not paying Shad, Inc.'s invoices on time would be the lowest cost source of capital, at a cost of 14.845% Answer (A) is incorrect because the actual annual percentage rate based on forgoing Shad's discount is 14.845% This is lower than the rate on the bank loan (17.302% because it is a discount loan), or the cost of forgoing the discounts allowed by Fort Co (18.182%) and Riley Co (16.326%) Answer (B) is incorrect because the actual annual percentage rate based on forgoing Shad's discount is 14.845% This is lower than the rate on the bank loan (17.302% because it is a discount loan), or the cost of forgoing the discounts allowed by Fort Co (18.182%) and Riley Co (16.326%) Answer (C) is incorrect because the actual annual percentage rate based on forgoing Shad's discount is 14.845% This is lower than the rate on the bank loan (17.302% because it is a discount loan), or the cost of forgoing the discounts allowed by Fort Co (18.182%) and Riley Co (16.326%) 78 REQUIRED: The true statement about short-term interest rates DISCUSSION: (A) Historically, one facet of the term structure of interest rates (the relationship of yield and time to maturity) is that short-term interest rates have ordinarily been lower than long-term rates One reason is that less risk is involved in the short run Moreover, future expectations concerning interest rates affect the term structure Most economists believe that a long-term interest rate is an average of future expected short-term interest rates For this reason, the yield curve will slope upward if future rates are expected to rise, downward if interest rates are anticipated to fall, and remain flat if investors think the rate is stable Future inflation is incorporated into this relationship Another consideration is liquidity preference Investors in an uncertain world will accept lower rates on short-term investments because of their greater liquidity, whereas business debtors often prefer to pay higher rates on long-term debt to avoid the hazards of short-term maturities Answer (B) is incorrect because short-term rates are usually lower than long-term rates Answer (C) is incorrect because short-term rates are more likely to be greater than long-term rates if current levels of inflation are high Answer (D) is incorrect because long-term rates may be viewed as short-term rates adjusted by a risk factor 79 Answer (D) is correct The prime interest rate is the rate charged by commercial banks to their best (the largest and financially strongest) business customers It is traditionally the lowest rate charged by banks However, in recent years, banks have been making loans at still lower rates in response to competition from the commercial paper market Answer (A) is incorrect because the prime rate has nothing to with a commitment fee on a bank loan Answer (B) is incorrect because the effective rate on most companies' bank loans will be much higher than the prime rate Answer (C) is incorrect because the prime rate is a bank loan rate, not the rate on commercial paper 80 Answer (C) is correct When a firm borrows money from the bank, it is often required to keep a certain percentage of the funds in the bank at all times These compensating balances effectively increase the rate of interest on the money borrowed from the bank Answer (A) is incorrect because the floating interest rate is not always higher It should float up or down with the prime rate Answer (B) is incorrect because a restriction on a new issuance does not raise the interest rate on money previously borrowed Answer (D) is incorrect because, if a firm chooses not to use its full line of credit and is not charged for the unused portion, the rate of interest on the portion used does not increase 81 Answer (B) is correct Simple interest is charged on the amount actually paid to the borrower If interest is charged on a simple basis, the full $20,000 face value of the loan is made available to the borrower Answer (A) is incorrect because interest is charged on a discount basis when it is deducted from the face value borrowed Answer (C) is incorrect because interest charged on a discount basis results in a deduction from the face value The borrower does not receive the full face value of the loan Answer (D) is incorrect because interest is charged on an add-on basis when the face value of the loan initially equals the borrowed amount plus the nominal interest charge 82 Answer (B) is correct A minimum checking account balance that a firm must maintain with a commercial bank is a compensating balance A bank may require a borrower to keep a certain percentage of the face value of a loan in the firm's account This requirement raises the real rate of interest to the borrower Answer (A) is incorrect because the cash balance necessary for a firm to conduct day-to-day business is a transactions balance Answer (C) is incorrect because a cash balance held in reserve for random, unforeseen fluctuations in cash inflows and outflows is a precautionary balance Answer (D) is incorrect because a cash balance that is held to enable the firm to take advantage of any bargain purchases that might arise is a speculative balance 83 Answer (A) is correct Banks sometimes require a borrower to keep a certain percentage of the face amount of a loan in a non-interest-bearing checking account This requirement raises the effective rate of interest paid by the borrower This greater rate compensates a bank for services provided and results in greater profitability for the financial institution Answer (B) is incorrect because, in financial accounting, a valuation allowance is used to reflect losses on marketable securities Answer (C) is incorrect because a safety stock of inventory is held to avoid inventory stockouts Answer (D) is incorrect because large depositors may receive favorable treatment, but compensating balances are funds maintained by loan recipients for the benefit of the lender 84 Answer (C) is correct The discounted interest rate is based on the amount borrowed but is paid in advance It is calculated using the following formula: Interest Borrowed amount - interest Answer (A) is incorrect because it is the formula for the simple interest rate for a 1-year loan Answer (B) is incorrect because it is the formula for the add-on installment interest for a 1-year loan Answer (D) is incorrect because the correct formula for the discounted interest rate is interest divided by the borrowed amount less interest 85 Answer (C) is correct A time draft (trade acceptance) is a form of commercial draft because it is drawn by a seller on the buyer; that is, it calls for the buyer to pay a specified amount The draft and the shipping documents related to the goods are then sent to the buyer's bank, which transmits the draft to the buyer The buyer accepts the draft by signing it A time draft, however, is similar to a promissory note because it is payable at a specific time in the future rather than upon acceptance by the buyer, which is characteristic of a sight draft If a seller is reluctant to ship goods because of concern about the buyer's ability to pay a time draft, the seller's bank may, for a fee, guarantee payment This banker's acceptance is an assumption of the obligation to pay at the due date Answer (A) is incorrect because a sight draft calls for immediate payment upon delivery of the shipping documents to, and the acceptance of the draft by, the buyer Answer (B) is incorrect because an open account is a credit arrangement involving only the signing of an invoice by the buyer Answer (D) is incorrect because the description is of a conditional sales contract except that the seller, not the bank, retains title to the goods until the buyer has completed payment 86 Answer (D) is correct An unsecured loan is a loan made by a bank based on credit information about the borrower and the ability of the borrower to repay the obligation The loan is not secured by collateral, but is made on the signature of the borrower Unsecured credit is not backed by collateral Revolving credit, bankers' acceptances, lines of credit, and commercial paper are all unsecured means of borrowing Answer (A) is incorrect because a chattel mortgage is a loan secured by personal property (movable property such as equipment or livestock) Also, a floating lien is secured by property, such as inventory, the composition of which may be constantly changing Answer (B) is incorrect because a chattel mortgage is a loan secured by personal property (movable property such as equipment or livestock) Factoring is a form of financing in which receivables serve as security Answer (C) is incorrect because a chattel mortgage is a loan secured by personal property (movable property such as equipment or livestock) Also, a floating lien is secured by property, such as inventory, the composition of which may be constantly changing 87 Answer (C) is correct Commercial paper is the term for the short-term (typically less than months), unsecured, large denomination (often over $100,000) promissory notes issued by large, creditworthy companies to other companies and institutional investors In many instances, the maturity date is only a few days after issuance Answer (A) is incorrect because an agency security is issued by a corporation or agency created by the U.S government Examples are government securities issued by the bodies that finance mortgages, such as the Federal National Mortgage Association (Fannie Mae) Answer (B) is incorrect because bankers' acceptances are drafts drawn on deposits at a bank The acceptance by the bank guarantees payment at maturity They are normally used to finance a specific transaction Answer (D) is incorrect because a repurchase agreement involves a secured loan to a government securities dealer It allows the buyer to retain interest income although the seller-dealer can repurchase after a specified time 88 Answer (C) is correct Commercial paper is a form of unsecured note that is sold by only the most creditworthy companies It is issued at a discount from its face value and has a maturity period of 270 days or less Commercial paper usually carries a low interest rate in comparison to other means of financing SMA 4M, Understanding Financial Instruments, observes that no general (active) secondary market exists for commercial paper, but that "most dealers or organizations will repurchase an issue that they have sold." Answer (A) is incorrect because commercial paper usually has a maturity date of 270 days or less to avoid securities registration requirements Answer (B) is incorrect because commercial paper is often issued directly by the borrowing firm Answer (D) is incorrect because interest rates must be higher than those of Treasury bills to entice investors Commercial paper is more risky than Treasury bills 89 Answer (A) is correct Commercial paper is a form of unsecured note that is sold by only the most creditworthy companies It is issued at a discount from its face value and has a maturity period of less than 270 days Commercial paper usually carries a low interest rate in comparison to other means of financing Answer (B) is incorrect because only large companies with good credit ratings can find buyers for their commercial paper Answer (C) is incorrect because commercial paper is unsecured Answer (D) is incorrect because investors must pay a commission similar to that on other investment securities 90 REQUIRED: The item not an advantage of using commercial paper for short-term financing DISCUSSION: (C) Commercial payer is a short-term, unsecured note payable issued in large denominations by major companies with excellent credit ratings Maturities usually not exceed 270 days Commercial paper is a lower cost source of funds than bank loans, and no compensating balances are required Commercial paper provides a broad and efficient distribution of debt, and costly financing arrangements are avoided The market is not open to all companies because only major corporations with high credit ratings can participate Answers (A), (B) and (D) are incorrect because lower rates, avoidance of compensating balance requirements, and broad debt distribution are advantages of commercial paper 91 Statement a is incorrect, and therefore the right answer Commercial paper is a type of unsecured promissory note issued by large, strong firms Statements b, c, d, and e are all accurate statements 92 Answer (A) is correct A short-term security issued by a corporation or agency created by the U.S government, such as the Federal Housing Administration, is an agency security (agency issue) Among the largest issuers of agency securities (excluding the Treasury) are the Federal Home Loan Banks, the Federal National Mortgage Association (Fannie Mae), and the other entities that provide credit to farmers and home buyers Other issuers of home mortgage-backed securities include the Government National Mortgage Association (Ginnie Mae) and the Federal Home Loan Mortgage Corporation (Freddie Mac) Answer (B) is incorrect because bankers' acceptances are issued by commercial banks to finance specific transactions Answer (C) is incorrect because commercial paper is a short-term, unsecured, promissory note issued by a commercial enterprise Answer (D) is incorrect because a repurchase agreement involves what is in essence a secured loan to a dealer in government securities 93 Answer (B) is correct A document of title is usually issued by a bailee covering goods in the bailee's possession or care (UCC 1-201) It represents ownership of the goods and is ordinarily needed to obtain the goods from the bailee The two major types of documents of title are bills of lading (issued by carriers) and warehouse receipts A warehouse receipt is issued by a person engaged in the business of storing goods for hire Security for short-term inventory financing can be arranged if the debtor places its inventory under the control of the lender or its agent (e.g., a public warehouse), and the lender holds the warehouse receipts Answer (A) is incorrect because commercial paper is a type of unsecured, short-term promissory note issued by large firms to other firms, insurance companies, mutual funds, etc Answer (C) is incorrect because a revolving credit agreement is a formal line of credit, usually with a bank, that large firms often use Answer (D) is incorrect because a line of credit is an arrangement, which may be formal or informal, between a commercial bank and its customer concerning the maximum loan amount available 94 Answer (B) is correct The corporation can obtain trade credit for 20 additional days by not paying within the discount period Instead of paying $99,000 to satisfy its obligation within 10 days, it can pay $100,000 at the end of 30 days The corporation will thus incur $1,000 in interest to hold the $99,000 for the 20 days Because a 360-day year has 18 such periods, the interest rate is approximately 18.18% [($1,000/$99,000) x 18] However, if compounding effects are considered, the rate is higher The effective rate, taking compounding into consideration, is found using the following formula: Effective rate = [1 + (1,000/99,000)]18- 1.0 = 19.83% In comparison, the 30-day note has an effective annual rate of 21.94%, calculated as follows: [1 + (.20/12)]12 - 1.0 = 21.94% Therefore, the corporation should use trade credit to obtain the short-term credit Answer (A) is incorrect because the effective trade credit rate is 19.83% depending on the method of calculation Answer (C) is incorrect because the note has an effective rate, including compounding effects, of 21.94% The following is the calculation: Effective rate = [1 + (.20/12)]12- 1.0 = 21.94% Answer (D) is incorrect because the note has an effective rate, including compounding effects, of 21.94% The following is the calculation: Effective rate = [1 + (.20/12)]12- 1.0 = 21.94% 95 Answer (D) is correct The most costly combination of characteristics is a higher compensating balance and discount interest The higher the compensating balance, the higher the portion of the loan funds that must be left on deposit with the lender Hence, the interest paid is charged on a smaller amount of funds available to be used by the borrower, and the effective cost is higher Also, discount interest is deducted from the loan funds in advance, resulting in a further increase in the effective financing cost Answer (A) is incorrect because lower compensating balances and regular interest are less costly Answer (B) is incorrect because lower compensating balances and regular interest are less costly Answer (C) is incorrect because lower compensating balances and regular interest are less costly 96 Answer (A) is correct The most desirable set of terms are those that result in the lowest cost of borrowing Discount interest results in a higher effective borrowing cost than simple interest because the bank deducts interest in advance so the borrower receives less than the face value of the loan A compensating balance results in a higher effective borrowing cost because the compensating balance is an amount of cash that the firm is unable to use The cheapest terms, given that all options have the same nominal interest rate, will be simple interest with no compensating balance Answer (B) is incorrect because discount interest is disadvantageous to the borrower Answer (C) is incorrect because a compensating balance is disadvantageous to the borrower Answer (D) is incorrect because discount interest and a compensating balance are disadvantageous to the borrower 97 Answer (A) is correct For any given quoted nominal rate, the least frequent compounding is associated with the lowest effective annual percentage cost Annual compounding is less frequent than semiannual, quarterly, or monthly The term of the loan is not relevant to the calculation of the effective annual percentage cost of financing Answer (B) is incorrect because, the more frequent the interest compounding, the more costly the loan Semiannual, quarterly, and monthly compounding are all more frequent than annual compounding Answer (C) is incorrect because, the more frequent the interest compounding, the more costly the loan Semiannual, quarterly, and monthly compounding are all more frequent than annual compounding Answer (D) is incorrect because, the more frequent the interest compounding, the more costly the loan Semiannual, quarterly, and monthly compounding are all more frequent than annual compounding 98 Answer (D) is correct If the company chooses the line of credit, it will pay 17% interest on $80,000 ($100,000 $20,000 discount) and 1% on the $20,000 unused portion, a total of $13,800 The effective interest rate would thus be 17.25% ($13,800 ÷ $80,000) Answer (A) is incorrect because if the company forgoes the cash discount, its effective rate is 25% ($20,000 ÷ $80,000 immediate cash price) Answer (B) is incorrect because an 18% effective rate exceeds that on the line of credit Answer (C) is incorrect because the effective rate would be 18.75% [(15% x $100,000) ÷ $80,000 available funds] ... QUESTIONS Page of 39 MANAGEMENT ADVISORY SERVICES WORKING CAPITAL FINANCE WORKING CAPITAL MANAGEMENT Appropriate Level of Working Capital Determining the appropriate level of working capital for a... 18 of 39 MANAGEMENT ADVISORY SERVICES C The loans are transferred to new lenders D None of the above WORKING CAPITAL FINANCE B&M Banker’s Acceptance 85 The credit instrument known as a banker's... approach to working capital will result in all permanent assets being financed using long-term securities CMA 0691 1-10 Page 19 of 39 MANAGEMENT ADVISORY SERVICES WORKING CAPITAL FINANCE d The

Ngày đăng: 28/02/2018, 15:43

Từ khóa liên quan

Mục lục

  • WORKING CAPITAL POLICY

  • Conservative Working Capital Policy

  • Aggressive Working Capital Policy

  • Aggressive vs. Conservative Working Capital Policy

  • WORKING CAPITAL MANAGEMENT

  • Appropriate Level of Working Capital

  • Working Capital

    • Effect of Transactions on Working Capital

    • Comprehensive

      • Motives for Holding Cash

      • Methods of Accelerating Cash Collections

        • Maximizing Net Float

        • Lock-Box System

        • Draft

        • Automated Clearing House (ACH) Electronic Transfer

        • Wire Transfer

        • Concentration Banking

        • Comprehensive

        • Baumol’s Model of Cash Balances

          • Cash Conversion Cycle

          • Optimal Mix Between Cash and Short-term Investments

            • Comprehensive

            • MARKETABLE SECURITIES

            • Criteria

            • Treasury Bills

              • Repurchase Agreement

Tài liệu cùng người dùng

Tài liệu liên quan